- 42,746
- 10,472
The answer looks rather different. Strange that theta does not appear in the final equation for T.kuruman said:This problem is from the Indian National Physics Olympiad – 2020. A "tentative" solution appears here on page 15.
This is troubling:
"1. Angular momentum is conserved only about P."
but later:
"We now calculate the acceleration of P on the rod in the direction of the string"
So is P accelerating or fixed? But this could just be sloppy wording.